Friday, November 17, 2017

calculus - Evaluating the limit: $limlimits_{x to infty} sumlimits_{n=1}^{infty} (-1)^{n-1}frac{x^{2n-1}}{(2n)! log (2n)}$



How to evaluate the limit: $$\lim\limits_{x \to \infty} \sum\limits_{n=1}^{\infty} (-1)^{n-1}\frac{x^{2n-1}}{(2n)!\log (2n)}$$



I have tried to break the sum into even and odd $n$ and estimate each sum to be $\sim \frac{e^{x}}{\log (4x)}$ by estimating the summations in the range $\sum\limits_{x/c \le n \le cx} \frac{x^{4n}}{(4n+s)!\log (4n+s)}$ with the trivial bounds while $e^{-x}\sum\limits_{n > cx} \frac{x^{4n}}{(4n+s)!\log (4n+s)}$ and $e^{-x}\sum\limits_{0 \le n < x/c} \frac{x^{4n}}{(4n+s)!\log (4n+s)}$ both decay exponentially for arbitrary $c > 1$ as $x \to +\infty$ ($s=2,4$). The precise estimates I have used are similar to here.


However, I suppose stronger asymptotics will be required for calculating the limit of their difference. Any help/hint is appreciated. Thanks!


I am not much acquainted with the Laplace method or probabilistic interpretations (I'd appreciate it some references are mentioned, should the answer involve advanced tools like them.)



Machinato suggests that $f(z)=\sum_{n=2}^{\infty} \frac{(-1)^n z^{n-1}}{n! \log n}$ approaches $\log z$. This seems to be true! As evidence, here are plots of the real (red) and imaginary (blue) parts of $f(e^{2+i \theta})$, for $-\pi<\theta<\pi$. For comparison, the red and blue dashed lines are the real and imaginary parts of $\log z = 2+i \theta$. It's not clear from this image whether the limit holds for all $\theta$, but for $-\pi/2 < \theta < \pi/2$ the fit is very good.



enter image description here


I'm offering a bounty for a proof of this bizarre behavior. I imagine the precise formulation is that, for $\theta$ fixed in $(-\pi,\pi)$, we have $\lim_{R \to \infty} f(R e^{i \theta}) - \log R = i \theta$ but I'll certain accept other asymptotic results of a similar flavor. (For $\theta = \pi$, it is easy to see that $f(-R)$ grows faster than any power of $R$, so it can't mimic $\log R$.)


After some more experimentation, I suspect the limit only holds for $-\pi/2<\theta<\pi/2$. Here are plots of $\mathrm{Im}(f(r e^{i \theta}))$ for $\theta = \pi/3$ (first plot) and $2 \pi/3$ (second plot), with $0 < r < 10$. In each case, the dashed line is at $\theta$. Convergence looks great in the first one, terrible in the second.


enter image description here


enter image description here


Answer



EDITED. It suffices to prove the following claim.



Proposition. Assume that $f(z) = \sum_{n=2}^{\infty} a_n z^n$ has radius of convergence $R$ and define


$$ F(z) = \sum_{n=2}^{\infty} \frac{n a_n}{n-1} z^n = z \int_{0}^{z} \frac{f'(w)}{w} \, dw. $$



Then for all $|z| < R$, we have


$$ \sum_{n=2}^{\infty} \frac{a_n}{\log n} z^n = \int_{0}^{1} \int_{0}^{\infty} \frac{t^{s-1}}{\Gamma(s)} F(z e^{-t}) \, dtds. $$



Proof. The proof is a straightforward computation:


\begin{align*} \int_{0}^{1} \int_{0}^{\infty} \frac{t^{s-1}}{\Gamma(s)} F(z e^{-t}) \, dtds &= \sum_{n=2}^{\infty} \frac{n a_n}{n-1} z^n \int_{0}^{1} \int_{0}^{\infty} \frac{t^{s-1}}{\Gamma(s)} e^{-nt} \, dtds \\ &= \sum_{n=2}^{\infty} \frac{n a_n}{n-1} z^n \int_{0}^{1} \frac{ds}{n^s} \\ &= \sum_{n=2}^{\infty} \frac{a_n}{\log n} z^n. \end{align*}



In our case, we can set $f(z) = 1 - \cos z$ and consequently $F(z) = z \operatorname{Si}(z)$, where $\operatorname{Si}(z)$ is the sine integral. Then for real $x$, we have


\begin{align*} \sum_{n=1}^{\infty} \frac{(-1)^{n-1}}{(2n)!\log(2n)} x^{2n-1} &= \int_{0}^{1} \int_{0}^{\infty} \frac{t^{s-1}}{\Gamma(s)} e^{-t} \operatorname{Si}(x e^{-t}) \, dtds \\ &\hspace{3em} \xrightarrow[x\to\infty]{} \quad \frac{\pi}{2} \int_{0}^{1}\int_{0}^{\infty} \frac{t^{s-1}}{\Gamma(s)} e^{-t} \, dtds = \frac{\pi}{2} \end{align*}


where the limiting procedure can be easily justified by the dominated convergence theorem.


Similarly, setting $f(z) = e^{-z} - 1 + z$ gives $F(z) = z (\log z + \gamma + \Gamma(0,z))$ for $\operatorname{Re}(z) > 0$, where $\Gamma(s, z)$ is the upper incomplete gamma function. Plugging this back, we obtain


$$ \sum_{n=2}^{\infty} \frac{(-1)^{n}}{n!\log n} z^{n-1} = \log z + \gamma - \frac{1}{2} + \int_{0}^{1} \int_{0}^{\infty} \frac{t^{s-1}}{\Gamma(s)} e^{-t} \Gamma(0, z e^{-t}) \, dtds. $$



Note that the last integral vanishes if we let $z \to \infty$ along the cone $|\arg(z)| \leq \frac{\pi}{2} - \delta$. So this again confirms numerical observation made by Machinato.


No comments:

Post a Comment

analysis - Injection, making bijection

I have injection $f \colon A \rightarrow B$ and I want to get bijection. Can I just resting codomain to $f(A)$? I know that every function i...